Difference of Closed Convex sets in Banach Space












2












$begingroup$


Let $A$ be a closed, convex, set in a Banach space $X$, and let $B$ be a closed, bounded, convex set in $X$. Assume that $A cap B = emptyset$. Set $C = A- B$. Prove that $C$ is closed, and convex.



So proving $C$ is convex is not too hard, however I am having issues proving it is closed. According to existing convex geometry literature, this is a well-known result, that requires the assumption of $B$ being bounded. However, I am not sure how to use it. I assume the proof would begin by picking $c_n in C$ such that $c_n to c$, $c in X$, and showing that in fact that $c in C$. We would write:
$$
c_n = a_n - b_n
$$

and somehow show that $c = a -b$ for some $a,b in C$, presumably $a = lim_{n to infty} a_n$, $b = lim_{n to infty} b_n$. But the existence of the limit for $c$ says nothing about the limits of $a_n,b_n$ (it can be deduced that $a_n$ is norm bounded using the norm bound on $b_n$), but I am truly at a loss on how to proceed. A hint would be appreciated.










share|cite|improve this question









$endgroup$












  • $begingroup$
    Are you sure $X$ is a Banach space? Off the top of my head, I can come up with a proof for reflexive spaces, using the fact that the unit ball is weakly compact. I can also come up with a counterexample if $B$ is not bounded. However, for arbitrary Banach spaces I do not yet see the answer.
    $endgroup$
    – SmileyCraft
    Jan 19 at 18:52










  • $begingroup$
    So the book I am using (Brezis'-Functional Analysis) actually does this in the context of Hilbert spaces! I realize now that he perhaps would intend on me decomposing $c_n = P_A(c_n) - P_B(c_n)$ and then passing to the limit (not sure if this works either because it does not use boundedness). But if you have a more general proof (in reflexive spaces) I would by all means be happy to see it.
    $endgroup$
    – rubikscube09
    Jan 19 at 18:55
















2












$begingroup$


Let $A$ be a closed, convex, set in a Banach space $X$, and let $B$ be a closed, bounded, convex set in $X$. Assume that $A cap B = emptyset$. Set $C = A- B$. Prove that $C$ is closed, and convex.



So proving $C$ is convex is not too hard, however I am having issues proving it is closed. According to existing convex geometry literature, this is a well-known result, that requires the assumption of $B$ being bounded. However, I am not sure how to use it. I assume the proof would begin by picking $c_n in C$ such that $c_n to c$, $c in X$, and showing that in fact that $c in C$. We would write:
$$
c_n = a_n - b_n
$$

and somehow show that $c = a -b$ for some $a,b in C$, presumably $a = lim_{n to infty} a_n$, $b = lim_{n to infty} b_n$. But the existence of the limit for $c$ says nothing about the limits of $a_n,b_n$ (it can be deduced that $a_n$ is norm bounded using the norm bound on $b_n$), but I am truly at a loss on how to proceed. A hint would be appreciated.










share|cite|improve this question









$endgroup$












  • $begingroup$
    Are you sure $X$ is a Banach space? Off the top of my head, I can come up with a proof for reflexive spaces, using the fact that the unit ball is weakly compact. I can also come up with a counterexample if $B$ is not bounded. However, for arbitrary Banach spaces I do not yet see the answer.
    $endgroup$
    – SmileyCraft
    Jan 19 at 18:52










  • $begingroup$
    So the book I am using (Brezis'-Functional Analysis) actually does this in the context of Hilbert spaces! I realize now that he perhaps would intend on me decomposing $c_n = P_A(c_n) - P_B(c_n)$ and then passing to the limit (not sure if this works either because it does not use boundedness). But if you have a more general proof (in reflexive spaces) I would by all means be happy to see it.
    $endgroup$
    – rubikscube09
    Jan 19 at 18:55














2












2








2


0



$begingroup$


Let $A$ be a closed, convex, set in a Banach space $X$, and let $B$ be a closed, bounded, convex set in $X$. Assume that $A cap B = emptyset$. Set $C = A- B$. Prove that $C$ is closed, and convex.



So proving $C$ is convex is not too hard, however I am having issues proving it is closed. According to existing convex geometry literature, this is a well-known result, that requires the assumption of $B$ being bounded. However, I am not sure how to use it. I assume the proof would begin by picking $c_n in C$ such that $c_n to c$, $c in X$, and showing that in fact that $c in C$. We would write:
$$
c_n = a_n - b_n
$$

and somehow show that $c = a -b$ for some $a,b in C$, presumably $a = lim_{n to infty} a_n$, $b = lim_{n to infty} b_n$. But the existence of the limit for $c$ says nothing about the limits of $a_n,b_n$ (it can be deduced that $a_n$ is norm bounded using the norm bound on $b_n$), but I am truly at a loss on how to proceed. A hint would be appreciated.










share|cite|improve this question









$endgroup$




Let $A$ be a closed, convex, set in a Banach space $X$, and let $B$ be a closed, bounded, convex set in $X$. Assume that $A cap B = emptyset$. Set $C = A- B$. Prove that $C$ is closed, and convex.



So proving $C$ is convex is not too hard, however I am having issues proving it is closed. According to existing convex geometry literature, this is a well-known result, that requires the assumption of $B$ being bounded. However, I am not sure how to use it. I assume the proof would begin by picking $c_n in C$ such that $c_n to c$, $c in X$, and showing that in fact that $c in C$. We would write:
$$
c_n = a_n - b_n
$$

and somehow show that $c = a -b$ for some $a,b in C$, presumably $a = lim_{n to infty} a_n$, $b = lim_{n to infty} b_n$. But the existence of the limit for $c$ says nothing about the limits of $a_n,b_n$ (it can be deduced that $a_n$ is norm bounded using the norm bound on $b_n$), but I am truly at a loss on how to proceed. A hint would be appreciated.







functional-analysis convex-analysis






share|cite|improve this question













share|cite|improve this question











share|cite|improve this question




share|cite|improve this question










asked Jan 19 at 18:35









rubikscube09rubikscube09

1,270719




1,270719












  • $begingroup$
    Are you sure $X$ is a Banach space? Off the top of my head, I can come up with a proof for reflexive spaces, using the fact that the unit ball is weakly compact. I can also come up with a counterexample if $B$ is not bounded. However, for arbitrary Banach spaces I do not yet see the answer.
    $endgroup$
    – SmileyCraft
    Jan 19 at 18:52










  • $begingroup$
    So the book I am using (Brezis'-Functional Analysis) actually does this in the context of Hilbert spaces! I realize now that he perhaps would intend on me decomposing $c_n = P_A(c_n) - P_B(c_n)$ and then passing to the limit (not sure if this works either because it does not use boundedness). But if you have a more general proof (in reflexive spaces) I would by all means be happy to see it.
    $endgroup$
    – rubikscube09
    Jan 19 at 18:55


















  • $begingroup$
    Are you sure $X$ is a Banach space? Off the top of my head, I can come up with a proof for reflexive spaces, using the fact that the unit ball is weakly compact. I can also come up with a counterexample if $B$ is not bounded. However, for arbitrary Banach spaces I do not yet see the answer.
    $endgroup$
    – SmileyCraft
    Jan 19 at 18:52










  • $begingroup$
    So the book I am using (Brezis'-Functional Analysis) actually does this in the context of Hilbert spaces! I realize now that he perhaps would intend on me decomposing $c_n = P_A(c_n) - P_B(c_n)$ and then passing to the limit (not sure if this works either because it does not use boundedness). But if you have a more general proof (in reflexive spaces) I would by all means be happy to see it.
    $endgroup$
    – rubikscube09
    Jan 19 at 18:55
















$begingroup$
Are you sure $X$ is a Banach space? Off the top of my head, I can come up with a proof for reflexive spaces, using the fact that the unit ball is weakly compact. I can also come up with a counterexample if $B$ is not bounded. However, for arbitrary Banach spaces I do not yet see the answer.
$endgroup$
– SmileyCraft
Jan 19 at 18:52




$begingroup$
Are you sure $X$ is a Banach space? Off the top of my head, I can come up with a proof for reflexive spaces, using the fact that the unit ball is weakly compact. I can also come up with a counterexample if $B$ is not bounded. However, for arbitrary Banach spaces I do not yet see the answer.
$endgroup$
– SmileyCraft
Jan 19 at 18:52












$begingroup$
So the book I am using (Brezis'-Functional Analysis) actually does this in the context of Hilbert spaces! I realize now that he perhaps would intend on me decomposing $c_n = P_A(c_n) - P_B(c_n)$ and then passing to the limit (not sure if this works either because it does not use boundedness). But if you have a more general proof (in reflexive spaces) I would by all means be happy to see it.
$endgroup$
– rubikscube09
Jan 19 at 18:55




$begingroup$
So the book I am using (Brezis'-Functional Analysis) actually does this in the context of Hilbert spaces! I realize now that he perhaps would intend on me decomposing $c_n = P_A(c_n) - P_B(c_n)$ and then passing to the limit (not sure if this works either because it does not use boundedness). But if you have a more general proof (in reflexive spaces) I would by all means be happy to see it.
$endgroup$
– rubikscube09
Jan 19 at 18:55










1 Answer
1






active

oldest

votes


















4












$begingroup$

If $X$ is reflexive, then the theorem would hold. We find that $b_n$ must have a weak point of accumulation $b$ by the Banach-Alaoglu theorem and the Eberlein-Šmulian theorem. Then $bin B$ and $c+bin A$ by A convex subset of a Banach space is closed if and only if it is weakly closed and $c=c+b-b$.



That $B$ is bounded is really necessary for this theorem. Consider the following example.



Let $X=mathbb{R}^2$ and $A={(x,y):xgeq0,xygeq1}$ and $B={(x,y):xgeq0,xyleq-1}$. Then $A$ and $B$ are disjoint, closed, convex and bounded. However $A-B={(x,y):y>0}$ is not closed.



That $X$ is reflexive is also necessary for this theorem. Consider the following example.



Let $X=ell^1$ and $A={xin X:x_ngeq0,sum x_n=1}$. Define $Tin B(X)$ by $(Tx)_n=(1+frac1n)x_n$ and let $B=T(A)$. Then $A$ and $B$ are disjoint, closed, convex and bounded. So $0notin A-B$. However $e_n-T(e_n)to0$, so $A-B$ is not closed.



That $A$ and $B$ are convex is also necessary. Consider the following example.



Let $X=ell^2$ and let $A$ be the closed unit ball and let $B={(1+frac1n)e_n}$. Then $A$ and $B$ are disjoint, closed and bounded, and $A$ is convex. So $0notin A-B$. However $e_n-(1+frac1n)e_nto0$, so $A-B$ is not closed.



That $A$ and $B$ are convex is, however, not necessary in the finite dimensional case. We then find that $b_n$ must have a point of accumulation $b$ by the Bolzano-Weierstrass theorem. Then $bin B$ and $c+bin A$ and $c=c+b-b$.






share|cite|improve this answer











$endgroup$













  • $begingroup$
    Nice counterexample!
    $endgroup$
    – gerw
    Jan 19 at 19:44










  • $begingroup$
    Thank you, I understand the use of Alaoglu's theorem to extract a weak limit $b$, however would this have to phrased in the language of nets because compact $not implies$ sequentially compact in general?
    $endgroup$
    – rubikscube09
    Jan 19 at 19:48








  • 1




    $begingroup$
    @rubikscube09 Good point. I looked it up, and it follows from the Eberlein Smulian theorem.
    $endgroup$
    – SmileyCraft
    Jan 19 at 19:51






  • 1




    $begingroup$
    You have a bit of a typo, but that is a brilliant question worthy of its own post :)
    $endgroup$
    – SmileyCraft
    Jan 19 at 20:05






  • 1




    $begingroup$
    Nevermind, it's rather silly. Weakly closed implies strongly closed, so $mathrm{cl}(K) subset mathrm{cl-wk}(K)$ as $mathrm{cl-wk}(K)$ is a closed set containing $K$ . Meanwhile, strongly closed + convex implies weakly closed, so $mathrm{cl-wk}(K) subset mathrm{cl}(K)$ because $mathrm{cl}(K)$ is a weakly closed set containing $K$.
    $endgroup$
    – rubikscube09
    Jan 19 at 20:20











Your Answer





StackExchange.ifUsing("editor", function () {
return StackExchange.using("mathjaxEditing", function () {
StackExchange.MarkdownEditor.creationCallbacks.add(function (editor, postfix) {
StackExchange.mathjaxEditing.prepareWmdForMathJax(editor, postfix, [["$", "$"], ["\\(","\\)"]]);
});
});
}, "mathjax-editing");

StackExchange.ready(function() {
var channelOptions = {
tags: "".split(" "),
id: "69"
};
initTagRenderer("".split(" "), "".split(" "), channelOptions);

StackExchange.using("externalEditor", function() {
// Have to fire editor after snippets, if snippets enabled
if (StackExchange.settings.snippets.snippetsEnabled) {
StackExchange.using("snippets", function() {
createEditor();
});
}
else {
createEditor();
}
});

function createEditor() {
StackExchange.prepareEditor({
heartbeatType: 'answer',
autoActivateHeartbeat: false,
convertImagesToLinks: true,
noModals: true,
showLowRepImageUploadWarning: true,
reputationToPostImages: 10,
bindNavPrevention: true,
postfix: "",
imageUploader: {
brandingHtml: "Powered by u003ca class="icon-imgur-white" href="https://imgur.com/"u003eu003c/au003e",
contentPolicyHtml: "User contributions licensed under u003ca href="https://creativecommons.org/licenses/by-sa/3.0/"u003ecc by-sa 3.0 with attribution requiredu003c/au003e u003ca href="https://stackoverflow.com/legal/content-policy"u003e(content policy)u003c/au003e",
allowUrls: true
},
noCode: true, onDemand: true,
discardSelector: ".discard-answer"
,immediatelyShowMarkdownHelp:true
});


}
});














draft saved

draft discarded


















StackExchange.ready(
function () {
StackExchange.openid.initPostLogin('.new-post-login', 'https%3a%2f%2fmath.stackexchange.com%2fquestions%2f3079674%2fdifference-of-closed-convex-sets-in-banach-space%23new-answer', 'question_page');
}
);

Post as a guest















Required, but never shown

























1 Answer
1






active

oldest

votes








1 Answer
1






active

oldest

votes









active

oldest

votes






active

oldest

votes









4












$begingroup$

If $X$ is reflexive, then the theorem would hold. We find that $b_n$ must have a weak point of accumulation $b$ by the Banach-Alaoglu theorem and the Eberlein-Šmulian theorem. Then $bin B$ and $c+bin A$ by A convex subset of a Banach space is closed if and only if it is weakly closed and $c=c+b-b$.



That $B$ is bounded is really necessary for this theorem. Consider the following example.



Let $X=mathbb{R}^2$ and $A={(x,y):xgeq0,xygeq1}$ and $B={(x,y):xgeq0,xyleq-1}$. Then $A$ and $B$ are disjoint, closed, convex and bounded. However $A-B={(x,y):y>0}$ is not closed.



That $X$ is reflexive is also necessary for this theorem. Consider the following example.



Let $X=ell^1$ and $A={xin X:x_ngeq0,sum x_n=1}$. Define $Tin B(X)$ by $(Tx)_n=(1+frac1n)x_n$ and let $B=T(A)$. Then $A$ and $B$ are disjoint, closed, convex and bounded. So $0notin A-B$. However $e_n-T(e_n)to0$, so $A-B$ is not closed.



That $A$ and $B$ are convex is also necessary. Consider the following example.



Let $X=ell^2$ and let $A$ be the closed unit ball and let $B={(1+frac1n)e_n}$. Then $A$ and $B$ are disjoint, closed and bounded, and $A$ is convex. So $0notin A-B$. However $e_n-(1+frac1n)e_nto0$, so $A-B$ is not closed.



That $A$ and $B$ are convex is, however, not necessary in the finite dimensional case. We then find that $b_n$ must have a point of accumulation $b$ by the Bolzano-Weierstrass theorem. Then $bin B$ and $c+bin A$ and $c=c+b-b$.






share|cite|improve this answer











$endgroup$













  • $begingroup$
    Nice counterexample!
    $endgroup$
    – gerw
    Jan 19 at 19:44










  • $begingroup$
    Thank you, I understand the use of Alaoglu's theorem to extract a weak limit $b$, however would this have to phrased in the language of nets because compact $not implies$ sequentially compact in general?
    $endgroup$
    – rubikscube09
    Jan 19 at 19:48








  • 1




    $begingroup$
    @rubikscube09 Good point. I looked it up, and it follows from the Eberlein Smulian theorem.
    $endgroup$
    – SmileyCraft
    Jan 19 at 19:51






  • 1




    $begingroup$
    You have a bit of a typo, but that is a brilliant question worthy of its own post :)
    $endgroup$
    – SmileyCraft
    Jan 19 at 20:05






  • 1




    $begingroup$
    Nevermind, it's rather silly. Weakly closed implies strongly closed, so $mathrm{cl}(K) subset mathrm{cl-wk}(K)$ as $mathrm{cl-wk}(K)$ is a closed set containing $K$ . Meanwhile, strongly closed + convex implies weakly closed, so $mathrm{cl-wk}(K) subset mathrm{cl}(K)$ because $mathrm{cl}(K)$ is a weakly closed set containing $K$.
    $endgroup$
    – rubikscube09
    Jan 19 at 20:20
















4












$begingroup$

If $X$ is reflexive, then the theorem would hold. We find that $b_n$ must have a weak point of accumulation $b$ by the Banach-Alaoglu theorem and the Eberlein-Šmulian theorem. Then $bin B$ and $c+bin A$ by A convex subset of a Banach space is closed if and only if it is weakly closed and $c=c+b-b$.



That $B$ is bounded is really necessary for this theorem. Consider the following example.



Let $X=mathbb{R}^2$ and $A={(x,y):xgeq0,xygeq1}$ and $B={(x,y):xgeq0,xyleq-1}$. Then $A$ and $B$ are disjoint, closed, convex and bounded. However $A-B={(x,y):y>0}$ is not closed.



That $X$ is reflexive is also necessary for this theorem. Consider the following example.



Let $X=ell^1$ and $A={xin X:x_ngeq0,sum x_n=1}$. Define $Tin B(X)$ by $(Tx)_n=(1+frac1n)x_n$ and let $B=T(A)$. Then $A$ and $B$ are disjoint, closed, convex and bounded. So $0notin A-B$. However $e_n-T(e_n)to0$, so $A-B$ is not closed.



That $A$ and $B$ are convex is also necessary. Consider the following example.



Let $X=ell^2$ and let $A$ be the closed unit ball and let $B={(1+frac1n)e_n}$. Then $A$ and $B$ are disjoint, closed and bounded, and $A$ is convex. So $0notin A-B$. However $e_n-(1+frac1n)e_nto0$, so $A-B$ is not closed.



That $A$ and $B$ are convex is, however, not necessary in the finite dimensional case. We then find that $b_n$ must have a point of accumulation $b$ by the Bolzano-Weierstrass theorem. Then $bin B$ and $c+bin A$ and $c=c+b-b$.






share|cite|improve this answer











$endgroup$













  • $begingroup$
    Nice counterexample!
    $endgroup$
    – gerw
    Jan 19 at 19:44










  • $begingroup$
    Thank you, I understand the use of Alaoglu's theorem to extract a weak limit $b$, however would this have to phrased in the language of nets because compact $not implies$ sequentially compact in general?
    $endgroup$
    – rubikscube09
    Jan 19 at 19:48








  • 1




    $begingroup$
    @rubikscube09 Good point. I looked it up, and it follows from the Eberlein Smulian theorem.
    $endgroup$
    – SmileyCraft
    Jan 19 at 19:51






  • 1




    $begingroup$
    You have a bit of a typo, but that is a brilliant question worthy of its own post :)
    $endgroup$
    – SmileyCraft
    Jan 19 at 20:05






  • 1




    $begingroup$
    Nevermind, it's rather silly. Weakly closed implies strongly closed, so $mathrm{cl}(K) subset mathrm{cl-wk}(K)$ as $mathrm{cl-wk}(K)$ is a closed set containing $K$ . Meanwhile, strongly closed + convex implies weakly closed, so $mathrm{cl-wk}(K) subset mathrm{cl}(K)$ because $mathrm{cl}(K)$ is a weakly closed set containing $K$.
    $endgroup$
    – rubikscube09
    Jan 19 at 20:20














4












4








4





$begingroup$

If $X$ is reflexive, then the theorem would hold. We find that $b_n$ must have a weak point of accumulation $b$ by the Banach-Alaoglu theorem and the Eberlein-Šmulian theorem. Then $bin B$ and $c+bin A$ by A convex subset of a Banach space is closed if and only if it is weakly closed and $c=c+b-b$.



That $B$ is bounded is really necessary for this theorem. Consider the following example.



Let $X=mathbb{R}^2$ and $A={(x,y):xgeq0,xygeq1}$ and $B={(x,y):xgeq0,xyleq-1}$. Then $A$ and $B$ are disjoint, closed, convex and bounded. However $A-B={(x,y):y>0}$ is not closed.



That $X$ is reflexive is also necessary for this theorem. Consider the following example.



Let $X=ell^1$ and $A={xin X:x_ngeq0,sum x_n=1}$. Define $Tin B(X)$ by $(Tx)_n=(1+frac1n)x_n$ and let $B=T(A)$. Then $A$ and $B$ are disjoint, closed, convex and bounded. So $0notin A-B$. However $e_n-T(e_n)to0$, so $A-B$ is not closed.



That $A$ and $B$ are convex is also necessary. Consider the following example.



Let $X=ell^2$ and let $A$ be the closed unit ball and let $B={(1+frac1n)e_n}$. Then $A$ and $B$ are disjoint, closed and bounded, and $A$ is convex. So $0notin A-B$. However $e_n-(1+frac1n)e_nto0$, so $A-B$ is not closed.



That $A$ and $B$ are convex is, however, not necessary in the finite dimensional case. We then find that $b_n$ must have a point of accumulation $b$ by the Bolzano-Weierstrass theorem. Then $bin B$ and $c+bin A$ and $c=c+b-b$.






share|cite|improve this answer











$endgroup$



If $X$ is reflexive, then the theorem would hold. We find that $b_n$ must have a weak point of accumulation $b$ by the Banach-Alaoglu theorem and the Eberlein-Šmulian theorem. Then $bin B$ and $c+bin A$ by A convex subset of a Banach space is closed if and only if it is weakly closed and $c=c+b-b$.



That $B$ is bounded is really necessary for this theorem. Consider the following example.



Let $X=mathbb{R}^2$ and $A={(x,y):xgeq0,xygeq1}$ and $B={(x,y):xgeq0,xyleq-1}$. Then $A$ and $B$ are disjoint, closed, convex and bounded. However $A-B={(x,y):y>0}$ is not closed.



That $X$ is reflexive is also necessary for this theorem. Consider the following example.



Let $X=ell^1$ and $A={xin X:x_ngeq0,sum x_n=1}$. Define $Tin B(X)$ by $(Tx)_n=(1+frac1n)x_n$ and let $B=T(A)$. Then $A$ and $B$ are disjoint, closed, convex and bounded. So $0notin A-B$. However $e_n-T(e_n)to0$, so $A-B$ is not closed.



That $A$ and $B$ are convex is also necessary. Consider the following example.



Let $X=ell^2$ and let $A$ be the closed unit ball and let $B={(1+frac1n)e_n}$. Then $A$ and $B$ are disjoint, closed and bounded, and $A$ is convex. So $0notin A-B$. However $e_n-(1+frac1n)e_nto0$, so $A-B$ is not closed.



That $A$ and $B$ are convex is, however, not necessary in the finite dimensional case. We then find that $b_n$ must have a point of accumulation $b$ by the Bolzano-Weierstrass theorem. Then $bin B$ and $c+bin A$ and $c=c+b-b$.







share|cite|improve this answer














share|cite|improve this answer



share|cite|improve this answer








edited Jan 19 at 19:51

























answered Jan 19 at 19:24









SmileyCraftSmileyCraft

3,591517




3,591517












  • $begingroup$
    Nice counterexample!
    $endgroup$
    – gerw
    Jan 19 at 19:44










  • $begingroup$
    Thank you, I understand the use of Alaoglu's theorem to extract a weak limit $b$, however would this have to phrased in the language of nets because compact $not implies$ sequentially compact in general?
    $endgroup$
    – rubikscube09
    Jan 19 at 19:48








  • 1




    $begingroup$
    @rubikscube09 Good point. I looked it up, and it follows from the Eberlein Smulian theorem.
    $endgroup$
    – SmileyCraft
    Jan 19 at 19:51






  • 1




    $begingroup$
    You have a bit of a typo, but that is a brilliant question worthy of its own post :)
    $endgroup$
    – SmileyCraft
    Jan 19 at 20:05






  • 1




    $begingroup$
    Nevermind, it's rather silly. Weakly closed implies strongly closed, so $mathrm{cl}(K) subset mathrm{cl-wk}(K)$ as $mathrm{cl-wk}(K)$ is a closed set containing $K$ . Meanwhile, strongly closed + convex implies weakly closed, so $mathrm{cl-wk}(K) subset mathrm{cl}(K)$ because $mathrm{cl}(K)$ is a weakly closed set containing $K$.
    $endgroup$
    – rubikscube09
    Jan 19 at 20:20


















  • $begingroup$
    Nice counterexample!
    $endgroup$
    – gerw
    Jan 19 at 19:44










  • $begingroup$
    Thank you, I understand the use of Alaoglu's theorem to extract a weak limit $b$, however would this have to phrased in the language of nets because compact $not implies$ sequentially compact in general?
    $endgroup$
    – rubikscube09
    Jan 19 at 19:48








  • 1




    $begingroup$
    @rubikscube09 Good point. I looked it up, and it follows from the Eberlein Smulian theorem.
    $endgroup$
    – SmileyCraft
    Jan 19 at 19:51






  • 1




    $begingroup$
    You have a bit of a typo, but that is a brilliant question worthy of its own post :)
    $endgroup$
    – SmileyCraft
    Jan 19 at 20:05






  • 1




    $begingroup$
    Nevermind, it's rather silly. Weakly closed implies strongly closed, so $mathrm{cl}(K) subset mathrm{cl-wk}(K)$ as $mathrm{cl-wk}(K)$ is a closed set containing $K$ . Meanwhile, strongly closed + convex implies weakly closed, so $mathrm{cl-wk}(K) subset mathrm{cl}(K)$ because $mathrm{cl}(K)$ is a weakly closed set containing $K$.
    $endgroup$
    – rubikscube09
    Jan 19 at 20:20
















$begingroup$
Nice counterexample!
$endgroup$
– gerw
Jan 19 at 19:44




$begingroup$
Nice counterexample!
$endgroup$
– gerw
Jan 19 at 19:44












$begingroup$
Thank you, I understand the use of Alaoglu's theorem to extract a weak limit $b$, however would this have to phrased in the language of nets because compact $not implies$ sequentially compact in general?
$endgroup$
– rubikscube09
Jan 19 at 19:48






$begingroup$
Thank you, I understand the use of Alaoglu's theorem to extract a weak limit $b$, however would this have to phrased in the language of nets because compact $not implies$ sequentially compact in general?
$endgroup$
– rubikscube09
Jan 19 at 19:48






1




1




$begingroup$
@rubikscube09 Good point. I looked it up, and it follows from the Eberlein Smulian theorem.
$endgroup$
– SmileyCraft
Jan 19 at 19:51




$begingroup$
@rubikscube09 Good point. I looked it up, and it follows from the Eberlein Smulian theorem.
$endgroup$
– SmileyCraft
Jan 19 at 19:51




1




1




$begingroup$
You have a bit of a typo, but that is a brilliant question worthy of its own post :)
$endgroup$
– SmileyCraft
Jan 19 at 20:05




$begingroup$
You have a bit of a typo, but that is a brilliant question worthy of its own post :)
$endgroup$
– SmileyCraft
Jan 19 at 20:05




1




1




$begingroup$
Nevermind, it's rather silly. Weakly closed implies strongly closed, so $mathrm{cl}(K) subset mathrm{cl-wk}(K)$ as $mathrm{cl-wk}(K)$ is a closed set containing $K$ . Meanwhile, strongly closed + convex implies weakly closed, so $mathrm{cl-wk}(K) subset mathrm{cl}(K)$ because $mathrm{cl}(K)$ is a weakly closed set containing $K$.
$endgroup$
– rubikscube09
Jan 19 at 20:20




$begingroup$
Nevermind, it's rather silly. Weakly closed implies strongly closed, so $mathrm{cl}(K) subset mathrm{cl-wk}(K)$ as $mathrm{cl-wk}(K)$ is a closed set containing $K$ . Meanwhile, strongly closed + convex implies weakly closed, so $mathrm{cl-wk}(K) subset mathrm{cl}(K)$ because $mathrm{cl}(K)$ is a weakly closed set containing $K$.
$endgroup$
– rubikscube09
Jan 19 at 20:20


















draft saved

draft discarded




















































Thanks for contributing an answer to Mathematics Stack Exchange!


  • Please be sure to answer the question. Provide details and share your research!

But avoid



  • Asking for help, clarification, or responding to other answers.

  • Making statements based on opinion; back them up with references or personal experience.


Use MathJax to format equations. MathJax reference.


To learn more, see our tips on writing great answers.




draft saved


draft discarded














StackExchange.ready(
function () {
StackExchange.openid.initPostLogin('.new-post-login', 'https%3a%2f%2fmath.stackexchange.com%2fquestions%2f3079674%2fdifference-of-closed-convex-sets-in-banach-space%23new-answer', 'question_page');
}
);

Post as a guest















Required, but never shown





















































Required, but never shown














Required, but never shown












Required, but never shown







Required, but never shown

































Required, but never shown














Required, but never shown












Required, but never shown







Required, but never shown







Popular posts from this blog

Mario Kart Wii

What does “Dominus providebit” mean?

Antonio Litta Visconti Arese